matheraum.de
Raum für Mathematik
Offene Informations- und Nachhilfegemeinschaft

Für Schüler, Studenten, Lehrer, Mathematik-Interessierte.
Hallo Gast!einloggen | registrieren ]
Startseite · Forum · Wissen · Kurse · Mitglieder · Team · Impressum
Forenbaum
^ Forenbaum
Status Mathe
  Status Schulmathe
    Status Primarstufe
    Status Mathe Klassen 5-7
    Status Mathe Klassen 8-10
    Status Oberstufenmathe
    Status Mathe-Wettbewerbe
    Status Sonstiges
  Status Hochschulmathe
    Status Uni-Analysis
    Status Uni-Lin. Algebra
    Status Algebra+Zahlentheo.
    Status Diskrete Mathematik
    Status Fachdidaktik
    Status Finanz+Versicherung
    Status Logik+Mengenlehre
    Status Numerik
    Status Uni-Stochastik
    Status Topologie+Geometrie
    Status Uni-Sonstiges
  Status Mathe-Vorkurse
    Status Organisatorisches
    Status Schule
    Status Universität
  Status Mathe-Software
    Status Derive
    Status DynaGeo
    Status FunkyPlot
    Status GeoGebra
    Status LaTeX
    Status Maple
    Status MathCad
    Status Mathematica
    Status Matlab
    Status Maxima
    Status MuPad
    Status Taschenrechner

Gezeigt werden alle Foren bis zur Tiefe 2

Navigation
 Startseite...
 Neuerdings beta neu
 Forum...
 vorwissen...
 vorkurse...
 Werkzeuge...
 Nachhilfevermittlung beta...
 Online-Spiele beta
 Suchen
 Verein...
 Impressum
Das Projekt
Server und Internetanbindung werden durch Spenden finanziert.
Organisiert wird das Projekt von unserem Koordinatorenteam.
Hunderte Mitglieder helfen ehrenamtlich in unseren moderierten Foren.
Anbieter der Seite ist der gemeinnützige Verein "Vorhilfe.de e.V.".
Partnerseiten
Dt. Schulen im Ausland: Mathe-Seiten:Weitere Fächer:

Open Source FunktionenplotterFunkyPlot: Kostenloser und quelloffener Funktionenplotter für Linux und andere Betriebssysteme
StartseiteMatheForenMathematik-WettbewerbeAufgabe #30
Foren für weitere Schulfächer findest Du auf www.vorhilfe.de z.B. Informatik • Physik • Technik • Biologie • Chemie
Forum "Mathematik-Wettbewerbe" - Aufgabe #30
Aufgabe #30 < Wettbewerbe < Schule < Mathe < Vorhilfe
Ansicht: [ geschachtelt ] | ^ Forum "Mathematik-Wettbewerbe"  | ^^ Alle Foren  | ^ Forenbaum  | Materialien

Aufgabe #30: Übungsaufgabe
Status: (Übungsaufgabe) Übungsaufgabe Status 
Datum: 15:46 So 27.03.2005
Autor: Hanno

Hallo an alle!

Es seien $2n$ positive, reelle Zahlen [mm] $a_1,a_2,...,a_n,b_1,b_2,...,b_n$ [/mm] gegeben. Zeige, dass dann
[mm] $\sqrt[n]{\produkt_{k=1}^{n} (a_k+b_k)}\geq\sqrt[n]{\produkt_{k=1}^{n} a_k}+\sqrt[n]{\produkt_{k=1}^{n} b_k}$ [/mm]
gilt.



Liebe Grüße,
Hanno

        
Bezug
Aufgabe #30: Ansatz
Status: (Frage) beantwortet Status 
Datum: 21:57 Di 12.04.2005
Autor: Wandfliese

Ich habe umgeformt wie ein Wahnsinniger!
Falls der Ansatz richtig ist (was ich bezweifle) möchte ich mal den eleganten Weg sehen ^^
Das war die Brute Force Methode ;-)

[mm] $\sqrt[n]{\produkt_{k=1}^{n} (a_k+b_k)}\geq\sqrt[n]{\produkt_{k=1}^{n} a_k}+\sqrt[n]{\produkt_{k=1}^{n} b_k}$ [/mm]
[mm] \produkt_{k=1}^{n} (a_k+b_k)\ge(\sqrt[n]{\produkt_{k=1}^{n} a_k}+\sqrt[n]{\produkt_{k=1}^{n} b_k})^{n} [/mm]
[mm] \produkt_{k=1}^{n} (a_k+b_k)\ge \summe_{i=0}^{n} \vektor{n \\ i}(\sqrt[n]{\produkt_{k=1}^{n} a_k})^{n-i}(\sqrt[n]{\produkt_{k=1}^{n} b_k})^{i} [/mm]
[mm] \produkt_{k=1}^{n} (a_k+b_k)\ge \summe_{i=0}^{n} \vektor{n \\ i}\produkt_{k=1}^{n} a_k( \bruch{\produkt_{k=1}^{n} b_k}{\produkt_{k=1}^{n} a_k})^{ \bruch{i}{n}} [/mm]
[mm] \produkt_{k=1}^{n} (a_k+b_k)\ge \summe_{i=0}^{n} \vektor{n \\ i}\produkt_{k=1}^{n} a_k(\produkt_{k=1}^{n} \bruch{b_k}{a_k})^{ \bruch{i}{n}} [/mm]
[mm] \produkt_{k=1}^{n} (a_k+b_k)\ge\produkt_{k=1}^{n} a_k \summe_{i=0}^{n} \vektor{n \\ i}( \wurzel[n]{\produkt_{k=1}^{n} \bruch{b_k}{a_k}})^{i} [/mm]
[mm] \produkt_{k=1}^{n} (a_k+b_k)\ge\produkt_{k=1}^{n} a_k \summe_{i=0}^{n} \vektor{n \\ i}( \produkt_{k=1}^{n}\wurzel[n]{ \bruch{b_k}{a_k}})^{i} [/mm]
[mm] \produkt_{k=1}^{n} (\bruch{a_k+b_k}{a_k})\ge\summe_{i=0}^{n} \vektor{n \\ i}( \produkt_{k=1}^{n}\wurzel[n]{ \bruch{b_k}{a_k}})^{i} [/mm]
[mm] \produkt_{k=1}^{n} (1+\bruch{b_k}{a_k})\ge\summe_{i=0}^{n} \vektor{n \\ i}( \produkt_{k=1}^{n}\wurzel[n]{ \bruch{b_k}{a_k}})^{i} [/mm]

Ok soweit wären wir jetzt muss man nur noch beweisen, das die Summe des Ausdrucks [mm] \summe_{i=0}^{n} \vektor{n \\ i}( \produkt_{k=1}^{n}\wurzel[n]{ \bruch{b_k}{a_k}})^{i} [/mm] immer kleiner ist als  [mm] \produkt_{k=1}^{n} (1+\bruch{b_k}{a_k}) [/mm] wer hätte das gedacht ^^

Das eigentliche Problem ist  [mm] \vektor{n \\ i} [/mm] in der Summe. Den größten Einfluß auf die [mm] \summe_{i=0}^{n} \vektor{n \\ i}( \produkt_{k=1}^{n}\wurzel[n]{ \bruch{b_k}{a_k}})^{i} [/mm] hat der Vektor wenn  [mm] \produkt_{k=1}^{n}\wurzel[n]{ \bruch{b_k}{a_k}}=1 [/mm] ist. Dann wär das Ergebnis der [mm] \summe_{i=0}^{n} \vektor{n \\ i}( \produkt_{k=1}^{n}\wurzel[n]{ \bruch{b_k}{a_k}})^{i}=2^{n} [/mm] (da gilt  [mm] \summe_{i=0}^{n} \vektor{n \\ i}=2^{n}) [/mm] und das Ergebnis des Ausdrucks ebenfalls [mm] \produkt_{k=1}^{n} (1+\bruch{b_k}{a_k})=2^{n}. [/mm]
Wenn [mm] \produkt_{k=1}^{n}\wurzel[n]{ \bruch{b_k}{a_k}}<1 [/mm] gilt logischerweise (ich setz das mal als trivial vorraus ^^)
[mm] \produkt_{k=1}^{n} (1+\bruch{b_k}{a_k})>\summe_{i=0}^{n} \vektor{n \\ i}( \produkt_{k=1}^{n}\wurzel[n]{ \bruch{b_k}{a_k}})^{i} [/mm]

Aber wie gesagt das ist ja nur ein Ansatz und was man bräuchte wär ne Formel für [mm] \summe_{i=0}^{n} \vektor{n \\ i}( \produkt_{k=1}^{n}\wurzel[n]{ \bruch{b_k}{a_k}})^{i} [/mm] aber das ist ja der Hauptbestandteil der Aufgabe ;-) würd mich nicht wundern wenn andere hier auch gescheitert sind.
Würd mich freuen zu erfahren wie man diesen Ansatz weiterführen könnte (wenn ich mich nicht total verrannt habe :-))

MFG und danke im vorraus Wandfliese


Bezug
                
Bezug
Aufgabe #30: Antwort
Status: (Antwort) fertig Status 
Datum: 21:46 Fr 15.04.2005
Autor: Teletubyyy

Hallo Wandfliese,

[respekt] für die Umformungen und den Lösungsansatz. Ich hab deine Umformungen überprüft und es gilt in der Tat:
[mm]\sqrt[n]{\produkt_{k=1}^{n} (a_k+b_k)}\geq\sqrt[n]{\produkt_{k=1}^{n} a_k}+\sqrt[n]{\produkt_{k=1}^{n} b_k} \ \ \gdw\ \ \produkt_{k=1}^{n} (1+\bruch{b_k}{a_k})\ge\summe_{i=0}^{n} \vektor{n \\ i}( \produkt_{k=1}^{n}\wurzel[n]{ \bruch{b_k}{a_k}})^{i}[/mm]

Den Fall, [mm] dass$\produkt_{k=1}^{n}\wurzel[n]{\bruch{b_k}{a_k}}=1$ [/mm] würde ich als weitestgehend trivial ansehen.

Allerdings ist in dem Fall:
[mm] $\produkt_{k=1}^{n}\wurzel[n]{ \bruch{b_k}{a_k}}<1$ [/mm]
Die Ungleichung für mich keinenfalls trivial! (Entweder du hast da nen Denkfehler oder ich hab grad ein Brett vorm Kopf ...)

Im übrigen kannst, nachdem du die Gleichheit gesondert betrachtet hast, deinen 'trivialen' Fall vorraussetzen. Denn es gilt immer:
[mm]\produkt_{k=1}^{n}\wurzel[n]{ \bruch{b_k}{a_k}}<1\,\,\,\,\vee\,\,\,\, \produkt_{k=1}^{n}\wurzel[n]{ \bruch{a_k}{b_k}}<1[/mm]
Und wegen Symmetriegründen in der Ausgangsgleichung können ja die Beiden Folgen [mm] a_k [/mm] und [mm] b_k [/mm] vertauscht werden. Also [mm]a_k \leftarrow b_k\ \ und\ \ b_k \leftarrow a_k[/mm], für alle k<n.

Den eleganten Weg kann dir vermutlich Hanno zeigen (Ich hab die Aufgabe noch nicht gelöst). Und meine Bemühungen mit HM<AM<GM<QM durch Abschätzungen zum Ziel zu führen sind bisher gescheitert. Denn immerhin lautet die aufgabe ja [mm]QM(c_n)\ge QM(a_n)+QM(b_n)[/mm] mit [mm] c_n:=a_n+b_n. [/mm]

Gruß Samuel

Bezug
        
Bezug
Aufgabe #30: Lösung
Status: (Frage) beantwortet Status 
Datum: 14:53 Fr 15.04.2005
Autor: Wandfliese

Ich hab mich nochmal mit der Aufgabe beschäftig und bin dann doch noch zu einer Lösung gekommen.

Meine bisherigen Umformungsschritte:
[mm] $\sqrt[n]{\produkt_{k=1}^{n} (a_k+b_k)}\geq\sqrt[n]{\produkt_{k=1}^{n} a_k}+\sqrt[n]{\produkt_{k=1}^{n} b_k}$ [/mm]
[mm] \produkt_{k=1}^{n} (a_k+b_k)\ge(\sqrt[n]{\produkt_{k=1}^{n} a_k}+\sqrt[n]{\produkt_{k=1}^{n} b_k})^{n} [/mm]
[mm] \produkt_{k=1}^{n} (a_k+b_k)\ge \summe_{i=0}^{n} \vektor{n \\ i}(\sqrt[n]{\produkt_{k=1}^{n} a_k})^{n-i}(\sqrt[n]{\produkt_{k=1}^{n} b_k})^{i} [/mm]
[mm] \produkt_{k=1}^{n} (a_k+b_k)\ge \summe_{i=0}^{n} \vektor{n \\ i}\produkt_{k=1}^{n} a_k( \bruch{\produkt_{k=1}^{n} b_k}{\produkt_{k=1}^{n} a_k})^{ \bruch{i}{n}} [/mm]
[mm] \produkt_{k=1}^{n} (a_k+b_k)\ge \summe_{i=0}^{n} \vektor{n \\ i}\produkt_{k=1}^{n} a_k(\produkt_{k=1}^{n} \bruch{b_k}{a_k})^{ \bruch{i}{n}} [/mm]
[mm] \produkt_{k=1}^{n} (a_k+b_k)\ge\produkt_{k=1}^{n} a_k \summe_{i=0}^{n} \vektor{n \\ i}( \wurzel[n]{\produkt_{k=1}^{n} \bruch{b_k}{a_k}})^{i} [/mm]
[mm] \produkt_{k=1}^{n} (a_k+b_k)\ge\produkt_{k=1}^{n} a_k \summe_{i=0}^{n} \vektor{n \\ i}( \produkt_{k=1}^{n}\wurzel[n]{ \bruch{b_k}{a_k}})^{i} [/mm]
[mm] \produkt_{k=1}^{n} (\bruch{a_k+b_k}{a_k})\ge\summe_{i=0}^{n} \vektor{n \\ i}( \produkt_{k=1}^{n}\wurzel[n]{ \bruch{b_k}{a_k}})^{i} [/mm]
[mm] \produkt_{k=1}^{n} (1+\bruch{b_k}{a_k})\ge\summe_{i=0}^{n} \vektor{n \\ i}( \produkt_{k=1}^{n}\wurzel[n]{ \bruch{b_k}{a_k}})^{i} [/mm]
Jetzt kommt der neue Teil:
[mm] \produkt_{k=1}^{n} (1+\bruch{b_k}{a_k})\ge\summe_{i=0}^{n} \vektor{n \\ i}( \produkt_{k=1}^{n}\wurzel[n]{ \bruch{b_k}{a_k}})^{i}*1^{n-i} [/mm]
[mm] \produkt_{k=1}^{n} (1+\bruch{b_k}{a_k})\ge(1+\produkt_{k=1}^{n}\wurzel[n]{ \bruch{b_k}{a_k}})^{n} [/mm]
[mm] \wurzel[n]{\produkt_{k=1}^{n} (1+\bruch{b_k}{a_k})}\ge1+\produkt_{k=1}^{n}\wurzel[n]{ \bruch{b_k}{a_k}} [/mm]
[mm] \wurzel[n]{\produkt_{k=1}^{n} (1+\bruch{b_k}{a_k})}-\wurzel[n]{\produkt_{k=1}^{n} \bruch{b_k}{a_k}}\ge1 [/mm]

Da
[mm] \wurzel[n]{\produkt_{k=1}^{n} (1+\bruch{b_k}{a_k})}\ge\wurzel[n]{\produkt_{k=1}^{n} \bruch{b_k}{a_k}}+1 [/mm] gilt
[mm] \Rightarrow \wurzel[n]{\produkt_{k=1}^{n} (1+\bruch{b_k}{a_k})}=\wurzel[n]{\produkt_{k=1}^{n} \bruch{b_k}{a_k}}+1+x [/mm]
wobei [mm] x\in\IR [/mm] und [mm] x\ge0 [/mm]
[mm] \Rightarrow\wurzel[n]{\produkt_{k=1}^{n} \bruch{b_k}{a_k}}+1+x-\wurzel[n]{\produkt_{k=1}^{n} \bruch{b_k}{a_k}}\ge1 [/mm]

[mm] \Rightarrow x\ge0 [/mm]
womit
[mm] $\sqrt[n]{\produkt_{k=1}^{n} (a_k+b_k)}\geq\sqrt[n]{\produkt_{k=1}^{n} a_k}+\sqrt[n]{\produkt_{k=1}^{n} b_k}$ [/mm]
unter den in der Aufgabe gegebenen Bedingung gilt :-)


Bezug
                
Bezug
Aufgabe #30: Antwort
Status: (Antwort) fertig Status 
Datum: 21:54 Fr 15.04.2005
Autor: Teletubyyy

Hallo Wandfliese,

Ich fürchte dein Beweis ist so nicht richtig. Du drehst dich nur im Kreis, nach dem Prinzip. A sei zu zeigen. A [mm] \Rightarrow [/mm] B; B [mm] \Rightarrow [/mm] A, womit A beweisen wäre ...


> Da
>  [mm]\wurzel[n]{\produkt_{k=1}^{n} (1+\bruch{b_k}{a_k})}\ge\wurzel[n]{\produkt_{k=1}^{n} \bruch{b_k}{a_k}}+1[/mm]
> gilt

soll gelten. Wenn du schon weist, dass diese Ungleichung gilt, wärst du schon mit der Aufgabe fertig, (da die Gleichung zur ursprünglichen äquivalent ist)


>  [mm]\Rightarrow \wurzel[n]{\produkt_{k=1}^{n} (1+\bruch{b_k}{a_k})}=\wurzel[n]{\produkt_{k=1}^{n} \bruch{b_k}{a_k}}+1+x[/mm]
>  
> wobei [mm]x\in\IR[/mm] und [mm]x\ge0[/mm]
>  [mm]\Rightarrow\wurzel[n]{\produkt_{k=1}^{n} \bruch{b_k}{a_k}}+1+x-\wurzel[n]{\produkt_{k=1}^{n} \bruch{b_k}{a_k}}\ge1[/mm]
>  
> [mm]\Rightarrow x\ge0[/mm]
>  womit
>  [mm]\sqrt[n]{\produkt_{k=1}^{n} (a_k+b_k)}\geq\sqrt[n]{\produkt_{k=1}^{n} a_k}+\sqrt[n]{\produkt_{k=1}^{n} b_k}[/mm]
> unter den in der Aufgabe gegebenen Bedingung gilt :-)
>  

Du führst nur folgerungen aus einer Vermutung aus. Wenn die erste Ungleichung gelten würden, würde auch die letzte gelten. Aber du übergehst den eigentlichen Beweis. Denn du weist ja nicht, ob die erste Ungleichung gilt!

Gruß Samuel

Bezug
                        
Bezug
Aufgabe #30: Irrtum
Status: (Mitteilung) Reaktion unnötig Status 
Datum: 00:59 Sa 16.04.2005
Autor: Wandfliese

mmh nicht das ich mich grundlegend geirrt habe

Das ist nämlich das erste Mal, dass ich mit dem Produktzeichen rechne.

[mm] \produkt_{k=1}^{n} (1+\bruch{b_k}{a_k})=(1+\bruch{b_n}{a_n})*(1+\bruch{b_{n-1}}{a_{n-1}})*(1+\bruch{b_{n-2}}{a_{n-2}})*...*(1+\bruch{b_2}{a_2})*(1+\bruch{b_1}{a_1}) [/mm]

Falls ich mich nicht geirrt habe gilt sehrwohl
[mm] \wurzel[n]{\produkt_{k=1}^{n} (1+\bruch{b_k}{a_k})}\ge\wurzel[n]{\produkt_{k=1}^{n} \bruch{b_k}{a_k}}+1 [/mm]

Wenn du dir diese Faktoren anguckst kann man sehr leicht erkennen, dass wenn man alle diese Faktoren mit einander multipliziert nichts anderes steht als
[mm] (1+\bruch{b_n}{a_n})*(1+\bruch{b_{n-1}}{a_{n-1}})*(1+\bruch{b_{n-2}}{a_{n-2}})*...*(1+\bruch{b_2}{a_2})*(1+\bruch{b_1}{a_1})=1+\ldots+\produkt_{k=1}^{n} (\bruch{b_k}{a_k}) [/mm]
Du siehst da bleibt immer was über, obwohl man das eigentlich wegfallen lassen könnte, weil für den Beweis ja [mm] \ge [/mm] gefordert wird.

Hier ein Beispiel für ein beliebig gewähltes k
da [mm] (1+\bruch{b_k}{a_k})(1+\bruch{b_{k-1}}{a_{k-1}})=1+\bruch{b_{k-1}}{a_{k-1}}+\bruch{b_k}{a_k}+\bruch{b_k}{a_k}*\bruch{b_{k-1}}{a_{k-1}}=1+x+\bruch{b_k*b_{k-1}}{a_k*a_{k-1}} [/mm]

Wobei [mm] x\in\IR [/mm] und [mm] x\ge0 [/mm] gilt

Man könnte so weiterverfahren und immer das x wegfallen lassen (kann man ja machen, da man das Produkt sogar noch verkleinert)

[mm] \produkt_{k=1}^{n} (1+\bruch{b_k}{a_k})>(\produkt_{k=2}^{n} (1+\bruch{b_k}{a_k}))*(1+\bruch{b_1}{a_1})>(1+\produkt_{k=1}^{n} \bruch{b_k}{a_k}) [/mm]

Ich hoffe das ist jetzt Beweis genug aber muss schon sagen du hast mich argh zweifeln und meinen PC um 0:30 hochfahren lassen omg werd ich morgen müde sein ;-)

MFG Wandfliese

P.S.: Mir fällt gerade auf, dass ich die n-te Wurzel gar nicht bedacht habe aber dazu hab ich heute keinen Nerv mehr :-)

Bezug
        
Bezug
Aufgabe #30: Lösung (nicht die einfachste)
Status: (Mitteilung) Reaktion unnötig Status 
Datum: 11:45 Sa 16.04.2005
Autor: moudi

Hallo Zusammen

Ich werde jetzt die Aufgabe auch einmal lösen. Dazu benutze ich folgendes "allgemein bekanntes" Thoerem, mit dem man viele Ungleichungen beweisen kann:

Sei f(x) eine auf einen Intervall I konvexe Funktion [mm] ($x\in I\Rightarrow f''(x)\geq0$). [/mm] Weiter seien [mm] $\lambda_i$ [/mm] positive Zahlen mit [mm] $\sum_i \lambda_i=1$ [/mm] und seien [mm] $x_i$ [/mm] Zahlen aus dem Intervall I.
Dann gilt [mm] $f(\sum_i \lambda_i x_i)\leq \sum_i \lambda_i f(x_i)$. [/mm]
(Zusatz ist [mm] $f''(x)\leq [/mm] 0$ auf dem Intervall I, gilt natürlich [mm] $f(\sum_i \lambda_i x_i)\geq \sum_i \lambda_i f(x_i)$.) [/mm]

(Zur Begründung dieses Theorems: Sei [mm] $P_i(x_i,f(x_i))$, [/mm] so kann man [mm] $\sum_i \lambda_i P_i$ [/mm] als gewichteten Schwepunkt der Punkte [mm] $P_i$ [/mm] interpretieren. Betrachtet man die Tangente an die Funktion in [mm] $\bar x=\sum_i \lambda_i x_i$ [/mm] (= x-Koordinate des Schwerpunkts), so liegen die Punkte [mm] P_i [/mm] alle oberhalb der Tangente, da f konvex ist. Dann liegt auch der gewichtete Schwerpunkt oberhalb der Tangente, was gerade der Inhalt des Theorems ist.)

Nun zum Beweis: Wir betrachten die Funktion
[mm] $f(t)=\left(\prod_i (1-t)a_i+t b_i\right)^{1/n}$ [/mm]
Ich zeige, dass [mm] $f''(t)\leq0$ [/mm] ist für [mm] $0\leq t\leq [/mm] 1$. Dann folgt mit [mm] $t_1=0, t_2=1$ [/mm] und [mm] $\lambda_1=\lambda_2=\frac [/mm] 12$ dass
[mm] $f(\lambda_1 t_1+\lambda_2 t_2)=f(1/2)\geq\frac12 f(0)+\frac12 [/mm] f(1)$ oder
[mm] $\left(\prod_i \frac12(a_i+b_i)\right)^{1/n}\geq\frac12\left(\prod_i a_i\right)^{1/n}+\frac12\left(\prod_i b_i\right)^{1/n}$ [/mm]
was äquivalent zur zu beweisenden Ungleichung ist.

Für n=1 ist die Ungleichung trivialer Weise richtig. Ich nehme daher [mm] $n\geq2$ [/mm] an.
Es bleibt [mm] $f''(t)\leq [/mm] 0$ zu zeigen.

Ableiten ergibt:
[mm]f'(t)=\frac 1n \left(\prod_i (1-t)a_i+t b_i\right)^{1/n-1}\cdot \sum_i(b_i-a_i)\prod_{j,\,j\neq i}(1-t)a_j+t b_j [/mm]

Nochmals ableiten:
[mm] $f''(t)=\frac 1n(\frac 1n-1)\left(\prod_i (1-t)a_i+t b_i\right)^{1/n-2}\cdot\left(\sum_i(b_i-a_i)\prod_{j,\,j\neq i}(1-t)a_j+t b_j\right)^2$ [/mm]
    [mm] $+\frac [/mm] 1n [mm] \left(\prod_i (1-t)a_i+t b_i\right)^{1/n-1}\sum_i(b_i-a_i)\sum_{j,\,j\neq i}(b_j-a_j)\prod_{k,\,k\neq i,j}(1-t)a_k+t b_k [/mm] \ [mm] (\mbox{an dieser Stelle brauche ich n>1})$ [/mm]


Jetzt umformen:
[mm] $f''(t)=\frac 1n(\frac [/mm] 1n [mm] -1)\left(\prod_i (1-t)a_i+t b_i\right)^{1/n}\cdot \left(\sum_i\frac{b_i-a_i}{(1-t)a_i+t b_i}\right)^2$ [/mm]
    [mm] $+\frac 1n\left(\prod_i (1-t)a_i+t b_i\right)^{1/n}\cdot \sum_i\frac{b_i-a_i}{(1-t)a_i+t b_i}\cdot \sum_{j,\,j\neq i}\frac{b_j-a_j}{(1-t)a_j+tb_j}$ [/mm]

[mm] $f''(t)=\frac 1n\left(\prod_i (1-t)a_i+t b_i\right)^{1/n} \left((\frac1n-1)\left(\sum_i\frac{b_i-a_i}{(1-t)a_i+t b_i}\right)^2+ \sum_i\frac{b_i-a_i}{(1-t)a_i+t b_i}\cdot \sum_{j,\,j\neq i}\frac{b_j-a_j}{(1-t)a_j+tb_j}\right)$ [/mm]

[mm] f''(t)=\frac 1n\left(\prod_i (1-t)a_i+t b_i\right)^{1/n} \left(\frac1n\left(\sum_i\frac{b_i-a_i}{(1-t)a_i+t b_i}\right)^2 +\sum_i\frac{b_i-a_i}{(1-t)a_i+t b_i}\cdot \sum_{j,\,j\neq i}\frac{b_j-a_j}{(1-t)a_j+tb_j}- \left(\sum_i\frac{b_i-a_i}{(1-t)a_i+t b_i}\right)^2\right) [/mm]

[mm] $f''(t)=\frac 1n\left(\prod_i (1-t)a_i+t b_i\right)^{1/n} \left(\frac1n\left(\sum_i\frac{b_i-a_i}{(1-t)a_i+t b_i}\right)^2 -\sum_i\left(\frac{b_i-a_i}{(1-t)a_i+t b_i}\right)^2\right) [/mm] $

[mm] $f''(t)=\underbrace{\left(\prod_i (1-t)a_i+t b_i\right)^{1/n}}_{\mbox{positiv}} \underbrace{\left(\frac1n\left(\sum_i\frac{b_i-a_i}{(1-t)a_i+t b_i}\right)^2 -\sum_i\left(\frac{b_i-a_i}{(1-t)a_i+t b_i}\right)^2\right)}_{(\ddagger)}$ [/mm]

Es bleibt zu zeigen, dass [mm] $(\ddagger)$ [/mm] nicht positiv ist.
Dazu definiere ich [mm] $x_i=\frac{b_i-a_i}{(1-t)a_i+tb_i}$ [/mm] und [mm] $\lambda_i=\frac [/mm] 1n$ und [mm] $g(x)=x^2$. [/mm]
Die Funktion g ist dann überall konvex und es gilt daher
[mm] $g(\sum_i\lambda_i x_i)\leq \sum_i\lambda_i g(x_i)$ [/mm] oder
[mm] $\left(\frac1n\sum_i x_i\right)^2\leq\sum_i\frac1n x_i^2$, [/mm] was nach Multiplikation mit n gleichbedeutend ist, dass [mm] $(\ddagger)$ [/mm] nicht positiv ist.

Man sieht, ich verwende das erwähnte Theorem gleich zweimal.

mfG Moudi

Bezug
Ansicht: [ geschachtelt ] | ^ Forum "Mathematik-Wettbewerbe"  | ^^ Alle Foren  | ^ Forenbaum  | Materialien


^ Seitenanfang ^
www.matheraum.de
[ Startseite | Forum | Wissen | Kurse | Mitglieder | Team | Impressum ]